Đến nội dung

the unknown nội dung

Có 210 mục bởi the unknown (Tìm giới hạn từ 10-06-2020)



Sắp theo                Sắp xếp  

#642791 hệ thặng dư

Đã gửi bởi the unknown on 29-06-2016 - 17:06 trong Số học

Chứng minh rằng với mọi n là số nguyên dương luôn tồn tạị $k \in \mathbb{N}$ thỏa mãn:
$$2^{n}|19^{k}-97$$

Bạn có thể tham khảo ở đây: http://diendantoanho...psilon-mathbbn/




#642594 $p+q=r(p-q)^n$

Đã gửi bởi the unknown on 28-06-2016 - 10:51 trong Số học

(AoPS) Giả sử rằng $p,q$ là các số nguyên tố sao cho $3\nmid p+q$ và $n,r$ là các số nguyên dương. Tìm tất cả các bộ bốn $(p,q,n,r)$ thỏa mãn $p+q=r(p-q)^n$.




#642436 $x^{4}+4=py^{4}.$

Đã gửi bởi the unknown on 27-06-2016 - 14:25 trong Số học

Tìm tất cả các số nguyên tố $p$ sao cho phương trình sau có nghiệm nguyên: $x^{4}+4=py^{4}.$

Bài toán này từng được thảo luận ở đây: http://diendantoanho...-nghiệm-nguyên/




#642381 Tìm tất cả các số nguyên dương lẻ $n$ sao cho $n\mid 15^n...

Đã gửi bởi the unknown on 27-06-2016 - 07:00 trong Số học

Mình nghĩ bài toán tổng quát sau đây cũng đúng: Cho $m$ là một số nguyên dương, $m>1$. Tìm tất cả các số nguyên dương lẻ $n$ để $n\mid (2^m-1)^n+1$.




#642325 Inequalities From 2016 Mathematical Olympiads

Đã gửi bởi the unknown on 26-06-2016 - 20:10 trong Bất đẳng thức - Cực trị

Bài 46 (JBMO). Cho $a,b,c $ là ba số thực dương. Chứng minh rằng
\[\frac{8}{(a+b)^2 + 4abc} + \frac{8}{(b+c)^2 + 4abc} + \frac{8}{(a+c)^2 + 4abc} + a^2 + b^2 + c ^2 \ge \frac{8}{a+3} + \frac{8}{b+3} + \frac{8}{c+3}.\]
 

Ta có: $\sum \frac{8}{(a+b)^2+4abc}+\sum a^2\geq \sum \frac{8}{(a+b)^2+c(a+b)^2}+\sum \frac{a^2+b^2}{2}\geq \sum (\frac{8}{(c+1)(a+b)^2}+\frac{(a+b)^2}{4})\geq \sum \frac{2\sqrt{2}}{\sqrt{a+1}}$

Bất đẳng thức chứng minh đúng bởi vì ta có đánh giá sau: $\frac{2\sqrt{2}}{\sqrt{a+1}}\geq \frac{8}{a+3}\Leftrightarrow (a+3)^2\geq 8a+8\Leftrightarrow (a-1)^2\geq 0$ ( luôn đúng ).

Và vì vậy nên: $\sum \frac{2\sqrt{2}}{\sqrt{a+1}}\geq \sum \frac{8}{a+3}$.

Vậy bất đẳng thức chứng minh đúng, đẳng thức xảy ra khi và chỉ khi $a=b=c=1$.




#642324 $N=\frac{(a-b)(b-c)(c-a)}{2}+2$

Đã gửi bởi the unknown on 26-06-2016 - 20:01 trong Số học

Bài toán: (JBMO 2016) Tìm tất cả bộ ba số nguyên $(a,b,c)$ để số: $N=\frac{(a-b)(b-c)(c-a)}{2}+2$

là một lũy thừa của $2016$.




#642279 MAX: $P=max[a,b,c]-min[a,b,c]$

Đã gửi bởi the unknown on 26-06-2016 - 16:44 trong Bất đẳng thức và cực trị

Cho a,b,c thực thỏa mãn: $a^3+b^3+c^3-3abc=2$ và $a+b+c=2$.

Tìm GTLN của: $P=max[a,b,c]-min[a,b,c]$

Ta có $a^3+b^3+c^3-3abc=(a+b+c)(a^2+b^2+c^2-ab-bc-ca)=2\Rightarrow a^2+b^2+c^2-ab-bc-ca=1$

Như vậy ta có: $\left\{\begin{matrix} a^2+b^2+c^2-ab-bc-ca=1 (1) \\ a+b+c=2\\ \end{matrix}\right.$.

Không mất tính tổng quát có thể giả sử $a=\max[a,b,c]$ và $c=\min[a,b,c]$.

Ta có $b=2-a-c$ và thay vào $(1)$ và biến đổi ta được $a^2+c^2+ac-2a-2b+1=0\Leftrightarrow 4(a^2+c^2+ac-2a-2c+1)=0\Leftrightarrow 3(a+c)^2-8(a+c)+(a-c)^2+4=0\Leftrightarrow 3(a+c-\frac{4}{3})^2+(a-c)^2-\frac{4}{3}=0\geq (a-c)^2-\frac{4}{3}$

Như vậy ta có: $(a-c)^2\leq \frac{4}{3}\Rightarrow a-c\leq \frac{2\sqrt{3}}{3}$.

Đẳng thức xảy ra khi và chỉ khi: $\left\{\begin{matrix} a+c=\frac{4}{3}\\ a-c=\frac{2\sqrt{3}}{3}\\ \end{matrix}\right. \Rightarrow a=\frac{2+\sqrt{3}}{3},b=\frac{2}{3}, c=\frac{2-\sqrt{3}}{3}$.

Do đó GTLN của $\max[a,b,c]-\min[a,b,c]$ là $\frac{2\sqrt{3}}{3}$, xảy ra khi $a=\frac{2+\sqrt{3}}{3},b=\frac{2}{3}, c=\frac{2-\sqrt{3}}{3}$ và các hoán vị.




#642174 $n\mid 2^n-8$

Đã gửi bởi the unknown on 25-06-2016 - 21:13 trong Số học

Chứng minh rằng tồn tại vô hạn số nguyên dương $n$ để: $n\mid 2^n-8$.

Spoiler




#642095 $x^5+y^5+1=(x+2)^5+(y-3)^5$

Đã gửi bởi the unknown on 25-06-2016 - 10:14 trong Số học

 

P/S: Có lẽ đề sai. 

Thực ra đề không sai đâu bạn, chỉ là đề gốc là chứng minh không tồn tại các số nguyên $x,y$ thôi  :D .

Có một cách làm khác sử dụng định lí Fermat nhỏ:

Ta có: $x^5+y^5+1\equiv x+y+1(\mod 5)$ và $(x+2)^5+(y-3)^5\equiv x+2+y-3=x+y-1 (\mod 5)$

Và như vậy ta có: $x+y+1\equiv x+y-1(\mod5)\Rightarrow 2\equiv 0(\mod5)$ ( vô lí) nên phương trình vô nghiệm.

 

Một bài toán khác: Giải phương trình nghiệm nguyên: $x^5+y^5-1=(x+2)^5+(y-3)^5$.

p.s




#642090 $x^5+y^5+1=(x+2)^5+(y-3)^5$

Đã gửi bởi the unknown on 25-06-2016 - 09:43 trong Số học

Giải phương trình nghiệm nguyên: $x^5+y^5+1=(x+2)^5+(y-3)^5$.




#641732 MAX: $M=\frac{1}{1+x^2}+\frac{1}...

Đã gửi bởi the unknown on 22-06-2016 - 14:43 trong Bất đẳng thức và cực trị

Cho x,y,z dương thỏa mãn: $xy+yz+zx=1$.

Tìm MAX của: $M=\frac{1}{1+x^2}+\frac{1}{1+y^2}+\frac{1}{1+z^2}$

Để ý rằng ta có bất đẳng thức quen thuộc: $8(x+y+z)(xy+yz+zx)\leq9(x+y)(y+z)(z+x)$. Thật vậy bất đẳng thức này tương đương với $\sum x(y-z)^2\geq 0$ luôn đúng do $x,y,z$ dương, kết hợp với giả thiết $xy+yz+zx=1$ ta có: $\frac{x+y+z}{(x+y)(y+z)(z+x)}\leq \frac{9}{8}$,

Như vậy ta có: $\sum \frac{1}{x^2+1}= \sum \frac{1}{x^2+xy+xz+yz}=\sum \frac{1}{(x+z)(x+y)}=\frac{2(x+y+z)}{(x+y)(y+z)(z+x)}\leq 2.\frac{9}{8}=\frac{9}{4}$.

Đẳng thức xảy ra khi và chỉ khi $x=y=z=\frac{1}{\sqrt{3}}$.




#641027 $\left\{\begin{matrix} x^{3}+x(y...

Đã gửi bởi the unknown on 18-06-2016 - 11:09 trong Phương trình - hệ phương trình - bất phương trình

Giải hệ phương trình: $$\left\{\begin{matrix} x^{3}+x(y-z)^{2}=2\\y^{3}+y(z-x)^{2}=30 \\z^{3}+z(x-y)^{2}=16 \end{matrix}\right.$$

Từ hệ phương trình ta có: $x^3+x(y-z)^2+y^3+y(x-z)^2=2(z^3+z(x-y)^2)\Leftrightarrow (x+y-2z)(x^2+y^2+z^2)=0$. Hơn nữa nếu $x^2+y^2+z^2=0$ thì $x=y=z=0$ không thỏa mãn hệ. Vậy $x+y-2z=0$ hay $z=\frac{x+y}{2}$ và thay vào hai phương trình đầu ta có được:

$\left\{\begin{matrix} x^3+x\frac{(x-y)^2}{4}=2\\ y^3+y\frac{(x-y)^2}{4}=30\\ \end{matrix}\right. \Rightarrow \left\{\begin{matrix} 4x^3+x(x-y)^2=8\\ 4y^3+y(x-y)^2=120\\ \end{matrix}\right. \Rightarrow 15(4x^3+x(x-y)^2)-(4y^3+y(x-y)^2)=0\Leftrightarrow (3x-y)(25x^2-2xy+5y^2)=0$.

Nếu $25x^2-2xy+5y^2=0$ suy ra $x=y=0$ nên vô lí.

Nếu $3x-y=0$ thì $4x^3+x(x-y)^2=8x^3=8\Rightarrow x=1\Rightarrow y=3\Rightarrow z=2$.

Vậy hệ phương trình có nghiệm duy nhất $(1;3;2)$.




#640568 $0<\frac{a_j-a_k}{1+a_ja_k}<\sqrt...

Đã gửi bởi the unknown on 15-06-2016 - 20:32 trong Số học

Cho 13 số thực $a_1,a_2,...a_{13}$ đôi một khác nhau. Chứng minh rằng tồn tại $a_j$ và $a_k,1\le j,k \le 13$ sao cho

$0<\frac{a_j-a_k}{1+a_ja_k}<\sqrt{\frac{2-\sqrt{3}}{2+\sqrt{3}}}$

Một cách làm hơi lương giác một tí  :D :

Đặt $a_i=tan x_i$ với $x\in (-\frac{\pi }{2},\frac{\pi}{2})$. không mất tính tổng quát ta giả sử $a_1<a_2<...<a_{13}$. Khi đó ta có $-\frac{\pi}{2}<x_1<x_2<x_3<...<x_{13}<\frac{\pi}{2}<x_1+\pi$. Khi đó dễ thấy đoạn $[x_1,x_1+\pi]$ chia thành 13 đoạn bởi $x_2,x_3,...,x_{13}$ nên tồn tại một đoạn có độ dài không vượt quá $\frac{\pi}{13}$. Xét hai trường hợp:

TH1: Nếu có đoạn $[x_{i-1},x_i]$ có độ dài nhỏ hơn $\frac{\pi}{13}$, khi đó ta có: $0<x_i-x_{i-1}<\frac{\pi}{12}\Rightarrow 0<tan(x_i-x_{i-1})<tan\frac{\pi}{12}$.

TH2: Nếu chỉ có đoạn $[x_{13},x_1+\pi]$ có đội dài bé hơn $\frac{\pi}{13}$, khi đó ta có: $0<x_1+\pi-x_{13}<\frac{\pi}{12}\Rightarrow 0<tan(x_1+\pi-x_{13})<tan{\frac{\pi}{12}}\Rightarrow 0<tan(x_{1}-x_{13})<tan{\frac{\pi}{12}}$.

Như vậy tồn tại hai số $i,j$ để $0<tan(x_i-x_j)<\frac{\pi}{12}$. Để ý rằng $tan(x_i-x_j)=\frac{a_i-a_j}{1+a_ia_j}$ và $tan\frac{\pi}{12}=\sqrt{\frac{2-\sqrt{3}}{2+\sqrt{3}}}$, từ đó suy ra điều phải chứng minh.

Spoiler




#640542 Chứng minh: $a^6$ chia 7 dư 1 (a>1)

Đã gửi bởi the unknown on 15-06-2016 - 17:11 trong Số học

Mình nghĩ không nên áp dụng quá máy móc công cụ như vậy. :v

Với $a$ không là bội của $7$ thì đặt $a=7k\pm r(r\in (1;2;3))$.

$a^{3}=343k^{3}\pm 147k^{2}+21kr^{2}\pm r^{3}\Rightarrow a^{3}=7l\pm -1(k,l\in \mathbb{Z})\Rightarrow ĐPCM$

 

Có một cách khác để chứng minh thay cho việc xét các giá trị của bạn ( bởi nếu với các số lớn hơn, ví dụ $2017$ chẳng hạn thì chắc thử hết mùa Euro chưa xong  :D ).

Xét các số $a,2a,3a,4a,5a,6a$ và $a$ là một số không chia hết cho $7$, khi đó các số này cũng không chia hết cho $7$ và không tồn tại hai số chia $7$ cùng số dư, thật vậy nếu có, giả sử là số $ma-na$ ($0<n<m<7$), và ta có $0<m-n<7$ nên $(m-n)a$ không chia hết cho $7$ nên sẽ vô lí. Mặt khác một số chia không chia hết cho $7$ chia $7$ chỉ nhận $6$ số dư, tức là $a,2a,3a,4a,5a,6a$ chia $7$ nhận $6$ số dư khác nhau từ $1$ đến $6$ ( hay nói nôm na là $a,2a,3a,4a,5a,6a$ là một hệ thặng dư đầy đủ $mod$ $7$, cái này học sau  :D ). Tức là $a.2a.3a.4a.5a.6a\equiv 1.2.3.4.5.6 (mod 7)\Rightarrow a^6\equiv 1(mod7)$. Vậy bài toán được chứng minh.

Bằng cách tương tự ta chứng minh được định lí Fermat và tổng quát hơn là định lí Euler. :D




#640530 Marathon số học Olympic

Đã gửi bởi the unknown on 15-06-2016 - 16:39 trong Số học

Bài 57 Cho $a_{1}=\frac{2}{3}$,$a_{n+1}=\frac{1}{2}(a_{n}+\frac{1}{3a_{n}}$

CMR $\frac{3}{3a_{n}^{2}-1}$ là số chính phương với mọi $n\in \mathbb{N}^{*}$

Bằng quy nạp ta chứng minh rằng $\frac{3}{3a_n^2-1}$ là số chính phương với mọi $n\in \mathbb{N}^{*}$. Thật vậy theo giả thiết ta có điều cần chứng minh hiển đúng với $n=1$. Giả sử rằng với $n\geq 1$ thì điều giả sử là đúng thì ta sẽ chứng minh cho trường hợp $n+1$.

Theo giả thiết ta có: $\frac{1}{a_n-a_{n+1}}=\frac{6a_n}{3a_n^2-1}$. Theo giả thiết quy nạp ta có: $\frac{3}{3a_n^2-1}$ là số chính phương nên có số $k$ nguyên dương để $\frac{3}{3a_n^2-1}=k^2$$\Rightarrow a_n^2=\frac{k^2+3}{3k^2}\Rightarrow a_n=\sqrt{\frac{k^2+3}{3k^2}}$.

Mặt khác dễ thấy $a_n$ là số hữu tỉ với mọi số nguyên dương $n$, do đó $\sqrt{3(k^2+3)}=3a_nk$ là một số hữu tỉ mà $3(k^2+3)$ là một số nguyên dương nên $3(k^2+3)$ là một số chính phương. Mặt khác $3\mid 3(k^2+3)\Rightarrow 9\mid 3(k^2+3)\Rightarrow 3\mid k^2+3$ và suy ra $\sqrt{\frac{k^2+3}{3}}=\frac{\sqrt{3(k^2+3)}}{3}$ là một số nguyên.

Vậy ta có $\frac{1}{a_n-a_{n+1}}=\frac{6a_n}{3a_n^2-1}=2a_n.k^2=2k\sqrt{\frac{k^2+3}{3}}$ là một số nguyên.

Mặt khác từ giả thiết ta có $6a_na_{n+1}=3a_n^2+1\Leftrightarrow 3(a_n-a_{n+1})^2=3a_{n+1}^2-1\Rightarrow \frac{3}{3a_{n+1}^2-1}=(\frac{1}{a_n-a_{n+1}})^2$ là một số chính phương. Do đó theo nguyên lí quy nạp, bài toán được chứng minh.

Bài 58: (IMO 1987)

Cho $n$ là một số nguyên, $n\geq 2$. Chứng minh rằng nếu $k^2+k+n$ là một số nguyên tố với mọi số nguyên $k$ thỏa mãn $0\leq k\leq \sqrt{\frac{n}{3}}$ thì $k^2+k+n$ cũng sẽ là một số nguyên tố với mọi số nguyên $k$ thỏa $0\leq k \leq n-2$.




#640508 Đề thi vào lớp 10 chuyên Lương Thế Vinh Đồng Nai năm 2016-2017

Đã gửi bởi the unknown on 15-06-2016 - 15:33 trong Tài liệu - Đề thi

Bài này hoán vị vòng quang nên bạn chỉ có thể giả sử b nằm giữa a và c chứ không thể như phần màu đỏ được

ừ nhỉ mình nhầm, cám ơn bạn đã nhắc.




#640472 MAX: F=$\frac{1}{5-2ab}+\frac{1}...

Đã gửi bởi the unknown on 15-06-2016 - 12:27 trong Bất đẳng thức và cực trị

Cho a,b,c không âm thỏa mãn: $\sum a^2=3$

Tìm MAX: F=$\frac{1}{5-2ab}+\frac{1}{5-2bc}+\frac{1}{5-2ca}$

Một hướng làm khác :) :

Với $x\in [0;\frac{3}{2}]$ thì $\frac{1}{5-2x}\leq \frac{2x^2-2x+3}{9}\Leftrightarrow (x-1)^2(3-2x)\geq 0$ (luôn đúng).

Mặt khác theo AM-GM thì để ý rằng $ab,bc,ca\in[0;\frac{3}{2}]$ nên do đó ta có:

$\sum \frac{1}{5-2ab}\leq \sum \frac{2a^2b^2-2ab+3}{9}= \frac{2}{9}(\sum a^2b^2-\sum ab)+1$.

Mặt khác ta có: $\sum a^2b^2\leq \frac{(ab+bc+ca)^2}{3}\leq \frac{(ab+bc+ca)(a^2+b^2+c^2)}{3}= \sum ab\Rightarrow \sum \frac{1}{5-2ab}\leq 1$.

Đẳng thức xảy ra khi và chỉ khi $a=b=c=1$.




#640434 $4\leq x^{2}+y^{2}\leq 2(\left | x...

Đã gửi bởi the unknown on 15-06-2016 - 09:48 trong Phương trình, hệ phương trình và bất phương trình

Bài này hơi vượt THCS đấy nhỉ, sơ qua ý tưởng là thế này  :D :

Xét đường tròn $(O,2)$ và bốn điểm $A(1;1),B(1;-1),C(-1;1),D(-1;-1)$. Khi đó ta xét bốn đường tròn tâm $A,B,C,D$ cùng bán kính là $\sqrt{2}$. Dễ chứng minh rằng một điểm $M(x,y)$ thỏa mãn đề bài thì phải nằm trên hoặc ngoài đường tròn $(O)$, nhưng lại nằm trong hoặc trên các đường tròn tâm $A,B,C,D$ và từ đó ta biểu diễn được tọa độ các điểm thỏa.




#640338 $k=\frac{a^{2}+b^{2}}{ab-1}...

Đã gửi bởi the unknown on 14-06-2016 - 20:29 trong Số học

Cho $a,b$ là các số nguyên dương thoả mãn $k=\frac{a^{2}+b^{2}}{ab-1}$ là số nguyên. Chứng minh rằng: $k=5$.

Một cách làm khác  :) :

GIả sử trong các cặp số $(a,b)$ nguyên dương thỏa thì xét cặp $(a,b)$ có tổng $a+b$ nhỏ nhất, và không mất tính tổng quát giả sử $a\geq b$. Khi đó xét phương trình ẩn $x$: $x^2-x.kb+b^2+k=0$. Hiển nhiên phương trình này có một nghiệm là $a$ và theo định lí Viet thì phương trình có một nghiệm là $t$ thỏa:

$\left\{\begin{matrix} t+a=kb\\ ta=b^2+k\\ \end{matrix}\right.$

Từ đó dễ dàng suy ra $t$ nguyên dương và do tính nhỏ nhất của tổng $a+b$ ta suy ra $t\geq a$, do đó $2t\geq kb\Rightarrow 2at\geq kab\geq kb^2\Rightarrow 2b^2+2k\geq kb^2\Rightarrow b^2\leq \frac{2k}{k-2}$. Mặt khác do AM-GM dễ dàng suy ra $k=\frac{a^2+b^2}{ab-1}\geq \frac{2ab}{ab-1}> 2$ nên $k\geq 3$, suy ra $b^2\leq 6$. Đến đây suy ra $b=2$ hoặc $b=1$, rồi sau đó thử các giá trị này vào phương trình ban đầu tìm $a$ và suy ra được $k=5$ (đpcm).$\blacksquare$

 

P.s: Bằng cách tương tự như trên ta giải quyết được một bài toán quen thuộc sau: Cho $a,b$ là các số nguyên dương sao cho $k=\frac{a^2+b^2}{ab+1}$ là số nguyên. Chứng minh $k$ là số chính phương.




#640112 Tồn tại tam giác có diện tích nhỏ hơn $\frac{h^2(4\pi -3...

Đã gửi bởi the unknown on 13-06-2016 - 20:22 trong Toán rời rạc

Giả sử trong mặt phẳng có $7$ điểm và không có $3$ điểm nào thẳng hàng. Gọi $h$ là độ dài đoạn thẳng lớn nhất trong các đoạn thẳng nối $2$ trong $7$ điểm đã cho. Chứng minh tồn tại một tam giác có đỉnh thuộc $7$ điểm đã cho và có diện tích nhỏ hơn $\frac{h^2(4\pi -3\sqrt{3})}{24}$.

 

Tổng quát: Liệu có thể tổng quát bài toán này trong trường hợp $n$ ($n\geq3$) điểm trên mặt phẳng?




#639839 $2^p+3^p=x^{y+1}$

Đã gửi bởi the unknown on 12-06-2016 - 15:36 trong Số học

Giải phương trình: $2^p+3^p=x^{y+1}$, trong đó $p$ là số nguyên tố và $x,y$ là các số nguyên dương.




#639795 Chứng minh tồn tại các số nguyên a, b, c sao cho: $0 < \left |...

Đã gửi bởi the unknown on 12-06-2016 - 11:17 trong Số học

3. Chứng minh tồn tại các số nguyên a, b, c sao cho:

$0 < \left | a + b\sqrt{2} + c\sqrt{3} \right | < \frac{1}{1000}$

 

Cho $c=0$, ta sẽ chứng minh tồn tại các số nguyên $a,b$ để $\left | a+b\sqrt{2} \right |<\frac{1}{1000}$. Ta sẽ chọn $a,b$ là các số nguyên trong khai triển $(\sqrt{2}-1)^{n}=a+b\sqrt{2}$. Hiển nhiên vì $0< \sqrt{2}-1< 1$ nên với $n$ đủ lớn thì ta sẽ có $0< (\sqrt{2}-1)^{n}< \frac{1}{1000}$, tức là $0< (a+b\sqrt{2})< \frac{1}{1000}$, tức là ta có thể chọn được các số $a,b,c$ ( đpcm ).

Spoiler

 



#639775 Cập nhật tình hình, thảo luận, chém gió về kì thi vào lớp 10 THPT

Đã gửi bởi the unknown on 12-06-2016 - 10:00 trong Góc giao lưu

có bà con nào thi 2 trường không?

Có đây và chế @O0NgocDuy0O nữa đấy  :D




#639614 M=$\frac{3a^{4}+3b^{4}+c^{3}+2...

Đã gửi bởi the unknown on 11-06-2016 - 17:34 trong Bất đẳng thức và cực trị

Cho 3 số thực dương a,b,c. Tìm giá trị nhỏ nhất của biểu thức :

M=$\frac{3a^{4}+3b^{4}+c^{3}+2}{(a+b+c)^{3}}$

Sử dụng bất đẳng thức AM-GM cho bộ $4$ số ta được: $a^4+a^4+a^4+1\geq 4\sqrt[4]{a^{12}}=4a^3$. Và tương tự ta cũng được $3b^4+1\geq 4b^3$. Do đó: $3a^4+3b^4+c^3+2\geq 4a^3+4b^3+c^3$.

Đến đây sử dụng BĐT Holder ta được: $(4a^3+4b^3+c^3)(1+1+2)(1+1+2)\geq 4(a+b+c)^3\Rightarrow \frac{4a^3+4b^3+c^3}{(a+b+c)^3}\geq \frac{1}{4}$.

Đẳng thức xảy ra khi $a=b=1,c=2$.

Vậy......




#639606 $\frac{27a^2}{c(c^2+9a^2)}+\frac{b^2...

Đã gửi bởi the unknown on 11-06-2016 - 16:56 trong Bất đẳng thức và cực trị

Bài toán: Cho ba số thực dương $a,b,c$ thỏa mãn $\frac{1}{a}+\frac{2}{b}+\frac{3}{c}=3$. Chứng minh:

$\frac{27a^2}{c(c^2+9a^2)}+\frac{b^2}{a(4a^2+b^2)}+\frac{8c^2}{b(9b^2+4c^2)}\geq \frac{3}{2}$